Evaluate using divergence theorem

In cylindrical coordinates, your volume integral is$$\iiint_V \,dV = \int_{z=3}^{z=4} \int_{\theta=0}^{2\pi} \int_{r=0}^{\sqrt{1-(z-3)^2}} r\,dr\,d\theta\,dz = \pi\int_3^4 1-(z-3)^2\,dz = \pi\int_3^4 (z^2-6z+9)\,dz = \frac{\pi}{3}.$$You can compare this to the answer you get when doing the surface integral, but as you can see, both
  • #1
DryRun
Gold Member
838
4
Homework Statement
http://s1.ipicture.ru/uploads/20120120/eAO1JUYk.jpg

The attempt at a solution
[tex]\int\int \vec{F}.\hat{n}\,ds=\int\int\int div\vec{F}\,dV[/tex]
where dV is the element of volume.
[tex]div\vec{F}=3[/tex]
Now, i need to find dV which (i assume) is the hardest part of this problem.
I've drawn the graph in my copybook and it's a paraboloid cut off by the plane z=3, with the outward unit normal vector pointing upward.

OK, i have 2 ideas for calculating the volume of that spherical shape bounded by the plane z=3:
1. I could use triple integral and transform to spherical coordinates. [itex]dxdydz={\rho}^2 \sin\phi .d\rho d\phi d\theta[/itex]
2. Use the flux formula for surface integral to find the surface area of the curved surface and then multiply it by the height along the z-axis which is 4-3=1.

I tried the first idea but i can't figure it out. So, I'm going with my second idea.
[tex]\phi(x,y,z)=z-4+x^2+y^2[/tex]
[tex]∇\vec{\phi}=2x\vec{i}+2y\vec{j}+\vec{k}[/tex]
[tex]\hat{n}=\frac{2x\vec{i}+2y\vec{j}+\vec{k}}{\sqrt{4x^2+4y^2+1}}[/tex]
[tex]Flux=\int\int \vec{F}.\hat{n}\,.d \sigma=\int\int \frac{2x^2+2y^2+z}{\sqrt{4 x^2+4y^2+1}}\,.d \sigma[/tex]
[tex]d\sigma = \sqrt{1+4x^2+4y^2}\,.dxdy[/tex]
[tex]S.A.=\int\int (2x^2+2y^2+z)\,.dxdy[/tex]
Finding the points of intersection of the plane z=3 and the paraboloid [itex]z=4-x^2-y^2[/itex], gives [itex]x^2+y^2=1[/itex] and hence r=1.

Transforming to polar coordinates:
[tex]S.A.=\int\int (2x^2+2y^2+z)\,.dxdy=\int\int (4+r^2)r\,.drd\theta=\int^{2\pi}_0 \int^1_0 (4r+r^3)\,.drd\theta=\frac{9\pi}{2}[/tex]
The volume becomes: [itex]\frac{9\pi}{2} \times 1 = \frac{9\pi}{2}[/itex]
[tex]\int\int\int div\vec{F}\,dV=\int\int\int 3\,dV=3\times \frac{9\pi}{2}=\frac{27\pi}{2}[/tex]
This is wrong, as (according to my notes) the correct answer is [itex]\frac{3\pi}{2}[/itex]. I have no idea what mistake/s i made.

EDIT: OK, after thinking more about this, i think i know where i made a mistake. I assumed the height of the paraboloid section to be 1 unit (at its peak), but the height is not constant, so i don't know how to find a value for the height. At this point, I'm thinking about going back to my first idea, which is finding the volume via triple integral using spherical coordinates. Any advice?

If i use triple integral with spherical coordinates:
[itex]z=4-x^2-y^2[/itex] becomes [itex]\rho\cos\phi=4-{\rho}^2 {\sin}^2 \phi[/itex]
This becomes: [itex]{\rho}^2 {\cos}^2 \phi +\rho \cos \phi=4+{\rho}^2[/itex]
It's a dilemma as I'm stuck with 2 variables.
 
Last edited:
Physics news on Phys.org
  • #2
Any help?
 
  • #3
I suggest evaluating the volume integral using cylindrical coordinates since the volume is symmetrical about the z-axis.
 
  • #4
sharks said:
Homework Statement
http://s1.ipicture.ru/uploads/20120120/eAO1JUYk.jpg

The attempt at a solution
[tex]\int\int \vec{F}.\hat{n}\,ds=\int\int\int div\vec{F}\,dV[/tex]
where dV is the element of volume.
[tex]div\vec{F}=3[/tex]
Now, i need to find dV which (i assume) is the hardest part of this problem.
I've drawn the graph in my copybook and it's a paraboloid cut off by the plane z=3, with the outward unit normal vector pointing upward.

OK, i have 2 ideas for calculating the volume of that spherical shape bounded by the plane z=3:
1. I could use triple integral and transform to spherical coordinates. [itex]dxdydz={\rho}^2 \sin\phi .d\rho d\phi d\theta[/itex]
2. Use the flux formula for surface integral to find the surface area of the curved surface and then multiply it by the height along the z-axis which is 4-3=1.

I tried the first idea but i can't figure it out. So, I'm going with my second idea.
[tex]\phi(x,y,z)=z-4+x^2+y^2[/tex]
[tex]∇\vec{\phi}=2x\vec{i}+2y\vec{j}+\vec{k}[/tex]
[tex]\hat{n}=\frac{2x\vec{i}+2y\vec{j}+\vec{k}}{\sqrt{4x^2+4y^2+1}}[/tex]
[tex]Flux=\int\int \vec{F}.\hat{n}\,.d \sigma=\int\int \frac{2x^2+2y^2+z}{\sqrt{4 x^2+4y^2+1}}\,.d \sigma[/tex]
[tex]d\sigma = \sqrt{1+4x^2+4y^2}\,.dxdy[/tex]
[tex]S.A.=\int\int (2x^2+2y^2+z)\,.dxdy[/tex]
Finding the points of intersection of the plane z=3 and the paraboloid [itex]z=4-x^2-y^2[/itex], gives [itex]x^2+y^2=1[/itex] and hence r=1.

Transforming to polar coordinates:
[tex]S.A.=\int\int (2x^2+2y^2+z)\,.dxdy=\int\int (4+r^2)r\,.drd\theta=\int^{2\pi}_0 \int^1_0 (4r+r^3)\,.drd\theta=\frac{9\pi}{2}[/tex]
Up to here, your work is fine. You've evaluated the surface integral over the upper surface of the solid, but you still need to add the contribution from the bottom surface. In other words, you have
$$\iint_S \vec{F}\cdot\hat{n}\,ds = \iint_{S_\text{top}} \vec{F}\cdot\hat{n}\,ds + \iint_{S_\text{bottom}} \vec{F}\cdot\hat{n}\,ds$$and you need to still do the second integral on the righthand side. The bottom surface is the unit circle lying in the z=3 plane centered about the z-axis.

The volume becomes: [itex]\frac{9\pi}{2} \times 1 = \frac{9\pi}{2}[/itex]
[tex]\int\int\int div\vec{F}\,dV=\int\int\int 3\,dV=3\times \frac{9\pi}{2}=\frac{27\pi}{2}[/tex]
This is wrong, as (according to my notes) the correct answer is [itex]\frac{3\pi}{2}[/itex]. I have no idea what mistake/s i made.
I'm not sure where you're getting the idea that the volume is 9π/2 times anything.

EDIT: OK, after thinking more about this, i think i know where i made a mistake. I assumed the height of the paraboloid section to be 1 unit (at its peak), but the height is not constant, so i don't know how to find a value for the height. At this point, I'm thinking about going back to my first idea, which is finding the volume via triple integral using spherical coordinates. Any advice?

If i use triple integral with spherical coordinates:
[itex]z=4-x^2-y^2[/itex] becomes [itex]\rho\cos\phi=4-{\rho}^2 {\sin}^2 \phi[/itex]
This becomes: [itex]{\rho}^2 {\cos}^2 \phi +\rho \cos \phi=4+{\rho}^2[/itex]
It's a dilemma as I'm stuck with 2 variables.
The point of the problem was to avoid doing the surface integrals by evaluating the volume integral, so this is indeed the tack you want to take. (Since you already did the hard part, you might want to finish off the surface integral calculation just to see it indeed all works out.) However, as I noted in my other post, cylindrical coordinates are better suited to this problem.
 
Last edited:
  • #5
sharks said:
Any help?
Before "bumping" your thread, you should wait 24 hours after you first post it. (It's a rule.)
 
  • #6
The instructions for this problem state that you are to use the divergence theorem. Therefore, you should evaluate [itex]\displaystyle {\int\!\!\!\!\int\!\!\!\!\int}_{D} \ div(\vec{F})\,dV\,. [/itex]

I agree with vela's suggestion to use cylindrical coordinates. After all, you're using the divergence theorem to make things easier. Why complicate things by using spherical coordinates? Cartesian coordinates would be easier to use than spherical coordinates.
 
  • #7
Thanks for the advice, vela. I'm going to find the flux of bottom disc.
SammyS, i apologize for bumping this topic a bit early. I waited 17 hours, but i should have read the rules.:redface:

Finding: [itex]\iint_{S_\text{bottom}} \vec{F}\cdot\hat{n}\,ds[/itex]
[tex]\phi(x,y,z)=-z+3[/tex][tex]∇\vec{\phi}=-\vec{k}[/tex][tex]\hat{n}=-\vec{k}[/tex][tex]Flux=\iint \vec{F}.\hat{n}\,.d \sigma=\iint -z\,.d \sigma[/tex][tex]d\sigma = \sqrt{0+0+1}\,.dxdy[/tex][tex]S.A.=\iint (-z)\,.dxdy=-\iint 3\,.dxdy[/tex]
Converting to polar coordinates:[tex]S.A.=-\int^{2\pi}_0 \int^1_0 3\,.rdrd\theta =-3\pi[/tex]
[tex]\iint_S \vec{F}\cdot\hat{n}\,ds = \iint_{S_\text{top}} \vec{F}\cdot\hat{n}\,ds + \iint_{S_\text{bottom}} \vec{F}\cdot\hat{n}\,ds=\frac{9\pi}{2}+-3\pi=\frac{3\pi}{2}[/tex]
This is correct. But now, I'm going to try to work it out as the problem intended using the divergence theorem.
[tex]\int\int \vec{F}.\hat{n}\,ds=\int\int\int div\vec{F}\,dV[/tex][tex]div\vec{F}=3[/tex]
Now, i have to find dV. To find the volume of this bell-shaped paraboloid using cylindrical coordinates is a bit odd for me, as i never used cylindrical coordinates for anything other than a perfect cylinder. I agree that the section under the plane z=3 will be as a cylinder, but I'm going for it with what i know. So, converting to cylindrical coordinates:
[tex]\int^{2\pi}_0 \int^1_0 \int^{4-r^2}_3 r.dzdrd\theta[/tex]
For θ and r fixed, z varies from z=3 to z=4-r^2
For θ fixed, r varies from 0 to 1
θ varies from 0 to 2∏
The answer is... [itex]\frac{\pi}{2}[/itex]
[tex]\int\int\int div\vec{F}\,dV=3\times \frac{\pi}{2}=\frac{3\pi}{2}[/tex] which is correct! Thanks, vela and SammyS for your advice.
 
Last edited:

Related to Evaluate using divergence theorem

1. What is the divergence theorem and why is it important in science?

The divergence theorem, also known as Gauss's theorem, relates a surface integral over a closed surface to a volume integral over the region inside the surface. This theorem is important in science because it allows us to convert a difficult surface integral into a more manageable volume integral, making it a powerful tool for solving problems in various fields such as physics, engineering, and mathematics.

2. How is the divergence theorem used in fluid mechanics?

In fluid mechanics, the divergence theorem is used to relate the flow of fluid through a surface to the sources and sinks of the fluid within the enclosed volume. This allows us to analyze and understand the behavior of fluids in complex systems, such as airflow over an airplane wing or the flow of blood through the human body.

3. Can the divergence theorem be applied to any type of vector field?

Yes, the divergence theorem can be applied to any continuous vector field, as long as the surface and volume integrals involved are well-defined. This makes it a versatile tool for a wide range of applications in science and engineering.

4. Is the divergence theorem related to other important theorems in mathematics?

Yes, the divergence theorem is closely related to other important theorems such as Stokes' theorem and Green's theorem. These theorems all fall under the branch of mathematics known as vector calculus, and they all relate different types of integrals to each other, making them powerful tools for solving a variety of problems.

5. Are there any limitations to using the divergence theorem?

One limitation of the divergence theorem is that it only applies to vector fields in three-dimensional space. Additionally, it may not be applicable in certain cases where the vector field is not continuous or the surface and volume integrals are not well-defined. In these cases, other mathematical tools may need to be used to solve the problem.

Similar threads

  • Calculus and Beyond Homework Help
Replies
3
Views
770
  • Calculus and Beyond Homework Help
Replies
4
Views
459
Replies
14
Views
1K
  • Calculus and Beyond Homework Help
Replies
20
Views
701
  • Calculus and Beyond Homework Help
Replies
6
Views
859
  • Calculus and Beyond Homework Help
Replies
9
Views
1K
  • Calculus and Beyond Homework Help
Replies
2
Views
975
  • Calculus and Beyond Homework Help
Replies
8
Views
1K
  • Calculus and Beyond Homework Help
Replies
5
Views
2K
  • Calculus and Beyond Homework Help
Replies
1
Views
705
Back
Top